Farmers who use genetically engineered plants on a large scale are at great financial risk because at any time a stud...

xDaltonLaney on November 6, 2019

Answer A

The same thing as a lot of these, I got stuck between A and B. On these 50/50 answer choices what's the best thing to do?

Reply
Create a free account to read and take part in forum discussions.

Already have an account? log in

Annie on November 6, 2019

Hi @xDaltonLaney,

If I understand your question correctly, you're asking what to do when you aren't sure between two different answer choices. Is this what you're asking? If so, here are my tips.

First, the most important thing to do is to answer the question and at least take your best guess. The LSAT has no penalty for wrong answers so you want to answer all of the questions, even with a guess. Second, the best hint I can give you when you're down to two answer choices is to look to the choices themselves and try to figure out what makes them different from one another. Once you pinpoint these differences, you can go back to the question with a fresh perspective and you may be able to find what you were missing before.

In this question, answer choices A and B are similar as, in a general sense, they both say that it may not be worth it for a farmer to grow genetically engineered crops. However, when you drill down into them the answer choices are quite different.

Answer A states that a farmer growing genetically engineered crops is taking a financial risk. This answer choice does not say that farmers should not grow these crops, but rather only says that doing so is risky.

Answer B states that it's not "prudent" for a farmer to grow genetically engineered crops. This answer choice goes farther than Answer A, and essentially says that it's a bad idea for farmers to grow these crops.

Now, it's time to turn back to the question. We are looking for the answer choice which rephrases the conclusion of the argument. The conclusion was that "it is unwise for farmers to grow such crops." As described above, Answer B says this in different words. Therefore, answer B is correct.